If you're seeing this message, it means we're having trouble loading external resources on our website.

If you're behind a web filter, please make sure that the domains *.kastatic.org and *.kasandbox.org are unblocked.

Main content

Course: LSAT > Unit 1

Lesson 5: Analytical Reasoning – Worked examples

Ordering setup | New info–could be true 3 | Worked example

Example video showing how to approach a "New info: could be true" question on an ordering setup from the analytical reasoning section of the LSAT.

Want to join the conversation?

No posts yet.

Video transcript

- [Instructor] Make sure that you've already watched the set-up video, in which we created this initial diagram, using the rules and making deductions. If M was begun in a later year than L, then which one of the following could be true? This question is giving us a new and temporary condition to consider the impact of. So if M was begun later than L, then which of the choices are possible in that scenario? So, we have our chain of elements already from the initial set-up, involving G, L, F, M and H. The new rule tells us that M is later than L. We know from the initial set-up, that M is before H. And that L is also before H. So that allows us to create a chain that's G, L, M, H in a row. And then from the initial set-up, we also know that L is earlier than F. So let's re-draw our initial diagram, so that we can incorporate this new rule and it's new deductions into a new sketch, and that way don't get confused with the initial diagram that we have. Alright, let's see what deductions we can make from the new information that we have. If M, is sometime later than L, which it is in this question, that means that only G and F could be the monuments that are paired in 601. Now we know that L has to be second, because in this question, L has three monuments after it, H, F and M. That means that M has to be third, because M can't be fourth or fifth according to rule three. And that leaves F and H to take up the fourth and fifth spots, although we don't know in which order. So our answer is something that could be true in this situation. A, F was begun in 603. Well this can't be true, because M is in 603. B, G was begun in 602. This can't be true, because we deduced that G has to be in 601. C tells that H was begun in 605, this could be true, because we deduced that either F or H could be in 605. For the record, choice D is that L was begun in 603, we know this is false because in this situation, L has to be in 602. And then E, is that S was begun in 604, and that must be false, because S is in 601. So, C is the answer.